Showing a sum is positiveFinding Binomial expansion of a radicalSimplify the Expression $sum _{ k=0 }^{ n }{...

Can I say "fingers" when referring to toes?

Is it allowed to activate the ability of multiple planeswalkers in a single turn?

Is there any evidence that Cleopatra and Caesarion considered fleeing to India to escape the Romans?

Does the Linux kernel need a file system to run?

Why is it that I can sometimes guess the next note?

Did the UK lift the requirement for registering SIM cards?

Were Persian-Median kings illiterate?

C++ check if statement can be evaluated constexpr

The IT department bottlenecks progress, how should I handle this?

Why should universal income be universal?

Strong empirical falsification of quantum mechanics based on vacuum energy density?

awk assign to multiple variables at once

What does Apple's new App Store requirement mean

Giving feedback to someone without sounding prejudiced

A variation to the phrase "hanging over my shoulders"

Why is the Sun approximated as a black body at ~ 5800 K?

Is this toilet slogan correct usage of the English language?

Has any country ever had 2 former presidents in jail simultaneously?

How would you translate "more" for use as an interface button?

What is the highest possible scrabble score for placing a single tile

What is going on with gets(stdin) on the site coderbyte?

Can I cause damage to electrical appliances by unplugging them when they are turned on?

Stack Interview Code methods made from class Node and Smart Pointers

Has the laser at Magurele, Romania reached a tenth of the Sun's power?



Showing a sum is positive


Finding Binomial expansion of a radicalSimplify the Expression $sum _{ k=0 }^{ n }{ binom{n}{k}}i^{k}3^{k-n} $Alternative combinatorial proof for $sumlimits_{r=0}^nbinom{n}{r}binom{m+r}{n}=sumlimits_{r=0}^nbinom{n}{r}binom{m}{r}2^r$Proof by induction, binomial coefficientApproximating a binomial sum over a simplexHow to expand $sqrt{x^6+1}$ using Maclaurin's seriesSum of ${m choose j}$ multiplied by $2^{2^j}$How to show that $sumlimits_{k=0}^n (-1)^ktfrac{{ {n}choose{k}}}{{ {x+k}choose{k}}} = frac{x}{x+n}$Finite sum with inverse binomialShowing an alternating sum is positive













2












$begingroup$



Show that the sum$$sum_{k=0}^{n} {n choose k}frac{{(-1)}^k}{n+k+1}$$ is a positive rational number.




It is easy to show that it is a rational number. But I am having trouble showing that this expression is positive. It might be some binomial expansion that I could not get.










share|cite|improve this question











$endgroup$








  • 1




    $begingroup$
    Have you tried using induction on $n$ for example?
    $endgroup$
    – Minus One-Twelfth
    1 hour ago


















2












$begingroup$



Show that the sum$$sum_{k=0}^{n} {n choose k}frac{{(-1)}^k}{n+k+1}$$ is a positive rational number.




It is easy to show that it is a rational number. But I am having trouble showing that this expression is positive. It might be some binomial expansion that I could not get.










share|cite|improve this question











$endgroup$








  • 1




    $begingroup$
    Have you tried using induction on $n$ for example?
    $endgroup$
    – Minus One-Twelfth
    1 hour ago
















2












2








2


1



$begingroup$



Show that the sum$$sum_{k=0}^{n} {n choose k}frac{{(-1)}^k}{n+k+1}$$ is a positive rational number.




It is easy to show that it is a rational number. But I am having trouble showing that this expression is positive. It might be some binomial expansion that I could not get.










share|cite|improve this question











$endgroup$





Show that the sum$$sum_{k=0}^{n} {n choose k}frac{{(-1)}^k}{n+k+1}$$ is a positive rational number.




It is easy to show that it is a rational number. But I am having trouble showing that this expression is positive. It might be some binomial expansion that I could not get.







combinatorics summation binomial-coefficients binomial-ideals






share|cite|improve this question















share|cite|improve this question













share|cite|improve this question




share|cite|improve this question








edited 1 hour ago







Hitendra Kumar

















asked 1 hour ago









Hitendra KumarHitendra Kumar

606




606








  • 1




    $begingroup$
    Have you tried using induction on $n$ for example?
    $endgroup$
    – Minus One-Twelfth
    1 hour ago
















  • 1




    $begingroup$
    Have you tried using induction on $n$ for example?
    $endgroup$
    – Minus One-Twelfth
    1 hour ago










1




1




$begingroup$
Have you tried using induction on $n$ for example?
$endgroup$
– Minus One-Twelfth
1 hour ago






$begingroup$
Have you tried using induction on $n$ for example?
$endgroup$
– Minus One-Twelfth
1 hour ago












3 Answers
3






active

oldest

votes


















4












$begingroup$

Direct proof:
$$begin{split}
sum_{k=0}^{n} {nchoose k}frac{{(-1)}^k}{n+k+1} &=sum_{k=0}^{n} {nchoose k}(-1)^kint_0^1 x^{n+k}dx\
&=int_0^1x^nsum_{k=0}^{n} {nchoose k}(-x)^kdx\
&=int_0^1x^n(1-x)^ndx
end{split}$$

The latter is clearly a positive number.






share|cite|improve this answer









$endgroup$













  • $begingroup$
    Thanks,I got it.
    $endgroup$
    – Hitendra Kumar
    45 mins ago










  • $begingroup$
    You're welcome!
    $endgroup$
    – Stefan Lafon
    44 mins ago










  • $begingroup$
    How did you conclude that the sum is a limited integral? Do you know where can I find more on this on-line? Thanks.
    $endgroup$
    – NoChance
    43 mins ago






  • 1




    $begingroup$
    It's a "known trick" that $frac 1 {p+1} = int_0^1x^pdx$. Then I noticed that the sum looked almost like that of the binomial theorem.
    $endgroup$
    – Stefan Lafon
    38 mins ago










  • $begingroup$
    Thanks for responding.Got it.
    $endgroup$
    – NoChance
    28 mins ago





















3












$begingroup$

When $k=0$ the term is positive. When $k=1$ the term is negative BUT SMALLER (in absolute value) THAN THE $k=0$ TERM.



When $k=2$ the term is positive. When $k=3$ the term is negative BUT SMALLER (in absolute value) THAN THE $k=2$ TERM.



.....



Get it?






share|cite|improve this answer









$endgroup$













  • $begingroup$
    sorry, I did not write question correctly. Now, I have corrected that. By looking at your answer I realized my mistake. Thanks
    $endgroup$
    – Hitendra Kumar
    1 hour ago



















0












$begingroup$

We can specifically prove that
$$
boxed{sum_{k=0}^{n} {n choose k}frac{{(-1)}^k}{n+k+1}=left((2n+1)binom{2n}nright)^{-1}}
$$

To see this, shuffle a deck of $2n+1$ cards numbered $1$ to $n$. Consider this:




What is the probability that card number $n+1$ is in the middle of the deck, and cards numbered $1$ to $n$ are below it?




The easy answer is the fraction on the RHS. The LHS can be interpreted as an application of the principle of inclusion exclusion. Namely, we first take the probability that card number of $n+1$ is the lowest of the cards numbered $n+1,n+2,dots,2n+1$. This is the $k=0$ term. From this, for each $i=1,dots,n$, we subtract the probability that $n+1$ is the lowest of the list $i,n+1,n+2,dots,2n+1$. This is a bad event, because we want $n+1$ to be above $i$. Doing this for each $i$, we subtract $binom{n}1frac{1}{n+2}$. We then must add back in the doubly subtracted events, subtract the triple intersections, and so on, eventually ending with the alternating sum on the left.






share|cite









$endgroup$













    Your Answer





    StackExchange.ifUsing("editor", function () {
    return StackExchange.using("mathjaxEditing", function () {
    StackExchange.MarkdownEditor.creationCallbacks.add(function (editor, postfix) {
    StackExchange.mathjaxEditing.prepareWmdForMathJax(editor, postfix, [["$", "$"], ["\\(","\\)"]]);
    });
    });
    }, "mathjax-editing");

    StackExchange.ready(function() {
    var channelOptions = {
    tags: "".split(" "),
    id: "69"
    };
    initTagRenderer("".split(" "), "".split(" "), channelOptions);

    StackExchange.using("externalEditor", function() {
    // Have to fire editor after snippets, if snippets enabled
    if (StackExchange.settings.snippets.snippetsEnabled) {
    StackExchange.using("snippets", function() {
    createEditor();
    });
    }
    else {
    createEditor();
    }
    });

    function createEditor() {
    StackExchange.prepareEditor({
    heartbeatType: 'answer',
    autoActivateHeartbeat: false,
    convertImagesToLinks: true,
    noModals: true,
    showLowRepImageUploadWarning: true,
    reputationToPostImages: 10,
    bindNavPrevention: true,
    postfix: "",
    imageUploader: {
    brandingHtml: "Powered by u003ca class="icon-imgur-white" href="https://imgur.com/"u003eu003c/au003e",
    contentPolicyHtml: "User contributions licensed under u003ca href="https://creativecommons.org/licenses/by-sa/3.0/"u003ecc by-sa 3.0 with attribution requiredu003c/au003e u003ca href="https://stackoverflow.com/legal/content-policy"u003e(content policy)u003c/au003e",
    allowUrls: true
    },
    noCode: true, onDemand: true,
    discardSelector: ".discard-answer"
    ,immediatelyShowMarkdownHelp:true
    });


    }
    });














    draft saved

    draft discarded


















    StackExchange.ready(
    function () {
    StackExchange.openid.initPostLogin('.new-post-login', 'https%3a%2f%2fmath.stackexchange.com%2fquestions%2f3157740%2fshowing-a-sum-is-positive%23new-answer', 'question_page');
    }
    );

    Post as a guest















    Required, but never shown

























    3 Answers
    3






    active

    oldest

    votes








    3 Answers
    3






    active

    oldest

    votes









    active

    oldest

    votes






    active

    oldest

    votes









    4












    $begingroup$

    Direct proof:
    $$begin{split}
    sum_{k=0}^{n} {nchoose k}frac{{(-1)}^k}{n+k+1} &=sum_{k=0}^{n} {nchoose k}(-1)^kint_0^1 x^{n+k}dx\
    &=int_0^1x^nsum_{k=0}^{n} {nchoose k}(-x)^kdx\
    &=int_0^1x^n(1-x)^ndx
    end{split}$$

    The latter is clearly a positive number.






    share|cite|improve this answer









    $endgroup$













    • $begingroup$
      Thanks,I got it.
      $endgroup$
      – Hitendra Kumar
      45 mins ago










    • $begingroup$
      You're welcome!
      $endgroup$
      – Stefan Lafon
      44 mins ago










    • $begingroup$
      How did you conclude that the sum is a limited integral? Do you know where can I find more on this on-line? Thanks.
      $endgroup$
      – NoChance
      43 mins ago






    • 1




      $begingroup$
      It's a "known trick" that $frac 1 {p+1} = int_0^1x^pdx$. Then I noticed that the sum looked almost like that of the binomial theorem.
      $endgroup$
      – Stefan Lafon
      38 mins ago










    • $begingroup$
      Thanks for responding.Got it.
      $endgroup$
      – NoChance
      28 mins ago


















    4












    $begingroup$

    Direct proof:
    $$begin{split}
    sum_{k=0}^{n} {nchoose k}frac{{(-1)}^k}{n+k+1} &=sum_{k=0}^{n} {nchoose k}(-1)^kint_0^1 x^{n+k}dx\
    &=int_0^1x^nsum_{k=0}^{n} {nchoose k}(-x)^kdx\
    &=int_0^1x^n(1-x)^ndx
    end{split}$$

    The latter is clearly a positive number.






    share|cite|improve this answer









    $endgroup$













    • $begingroup$
      Thanks,I got it.
      $endgroup$
      – Hitendra Kumar
      45 mins ago










    • $begingroup$
      You're welcome!
      $endgroup$
      – Stefan Lafon
      44 mins ago










    • $begingroup$
      How did you conclude that the sum is a limited integral? Do you know where can I find more on this on-line? Thanks.
      $endgroup$
      – NoChance
      43 mins ago






    • 1




      $begingroup$
      It's a "known trick" that $frac 1 {p+1} = int_0^1x^pdx$. Then I noticed that the sum looked almost like that of the binomial theorem.
      $endgroup$
      – Stefan Lafon
      38 mins ago










    • $begingroup$
      Thanks for responding.Got it.
      $endgroup$
      – NoChance
      28 mins ago
















    4












    4








    4





    $begingroup$

    Direct proof:
    $$begin{split}
    sum_{k=0}^{n} {nchoose k}frac{{(-1)}^k}{n+k+1} &=sum_{k=0}^{n} {nchoose k}(-1)^kint_0^1 x^{n+k}dx\
    &=int_0^1x^nsum_{k=0}^{n} {nchoose k}(-x)^kdx\
    &=int_0^1x^n(1-x)^ndx
    end{split}$$

    The latter is clearly a positive number.






    share|cite|improve this answer









    $endgroup$



    Direct proof:
    $$begin{split}
    sum_{k=0}^{n} {nchoose k}frac{{(-1)}^k}{n+k+1} &=sum_{k=0}^{n} {nchoose k}(-1)^kint_0^1 x^{n+k}dx\
    &=int_0^1x^nsum_{k=0}^{n} {nchoose k}(-x)^kdx\
    &=int_0^1x^n(1-x)^ndx
    end{split}$$

    The latter is clearly a positive number.







    share|cite|improve this answer












    share|cite|improve this answer



    share|cite|improve this answer










    answered 1 hour ago









    Stefan LafonStefan Lafon

    3,00019




    3,00019












    • $begingroup$
      Thanks,I got it.
      $endgroup$
      – Hitendra Kumar
      45 mins ago










    • $begingroup$
      You're welcome!
      $endgroup$
      – Stefan Lafon
      44 mins ago










    • $begingroup$
      How did you conclude that the sum is a limited integral? Do you know where can I find more on this on-line? Thanks.
      $endgroup$
      – NoChance
      43 mins ago






    • 1




      $begingroup$
      It's a "known trick" that $frac 1 {p+1} = int_0^1x^pdx$. Then I noticed that the sum looked almost like that of the binomial theorem.
      $endgroup$
      – Stefan Lafon
      38 mins ago










    • $begingroup$
      Thanks for responding.Got it.
      $endgroup$
      – NoChance
      28 mins ago




















    • $begingroup$
      Thanks,I got it.
      $endgroup$
      – Hitendra Kumar
      45 mins ago










    • $begingroup$
      You're welcome!
      $endgroup$
      – Stefan Lafon
      44 mins ago










    • $begingroup$
      How did you conclude that the sum is a limited integral? Do you know where can I find more on this on-line? Thanks.
      $endgroup$
      – NoChance
      43 mins ago






    • 1




      $begingroup$
      It's a "known trick" that $frac 1 {p+1} = int_0^1x^pdx$. Then I noticed that the sum looked almost like that of the binomial theorem.
      $endgroup$
      – Stefan Lafon
      38 mins ago










    • $begingroup$
      Thanks for responding.Got it.
      $endgroup$
      – NoChance
      28 mins ago


















    $begingroup$
    Thanks,I got it.
    $endgroup$
    – Hitendra Kumar
    45 mins ago




    $begingroup$
    Thanks,I got it.
    $endgroup$
    – Hitendra Kumar
    45 mins ago












    $begingroup$
    You're welcome!
    $endgroup$
    – Stefan Lafon
    44 mins ago




    $begingroup$
    You're welcome!
    $endgroup$
    – Stefan Lafon
    44 mins ago












    $begingroup$
    How did you conclude that the sum is a limited integral? Do you know where can I find more on this on-line? Thanks.
    $endgroup$
    – NoChance
    43 mins ago




    $begingroup$
    How did you conclude that the sum is a limited integral? Do you know where can I find more on this on-line? Thanks.
    $endgroup$
    – NoChance
    43 mins ago




    1




    1




    $begingroup$
    It's a "known trick" that $frac 1 {p+1} = int_0^1x^pdx$. Then I noticed that the sum looked almost like that of the binomial theorem.
    $endgroup$
    – Stefan Lafon
    38 mins ago




    $begingroup$
    It's a "known trick" that $frac 1 {p+1} = int_0^1x^pdx$. Then I noticed that the sum looked almost like that of the binomial theorem.
    $endgroup$
    – Stefan Lafon
    38 mins ago












    $begingroup$
    Thanks for responding.Got it.
    $endgroup$
    – NoChance
    28 mins ago






    $begingroup$
    Thanks for responding.Got it.
    $endgroup$
    – NoChance
    28 mins ago













    3












    $begingroup$

    When $k=0$ the term is positive. When $k=1$ the term is negative BUT SMALLER (in absolute value) THAN THE $k=0$ TERM.



    When $k=2$ the term is positive. When $k=3$ the term is negative BUT SMALLER (in absolute value) THAN THE $k=2$ TERM.



    .....



    Get it?






    share|cite|improve this answer









    $endgroup$













    • $begingroup$
      sorry, I did not write question correctly. Now, I have corrected that. By looking at your answer I realized my mistake. Thanks
      $endgroup$
      – Hitendra Kumar
      1 hour ago
















    3












    $begingroup$

    When $k=0$ the term is positive. When $k=1$ the term is negative BUT SMALLER (in absolute value) THAN THE $k=0$ TERM.



    When $k=2$ the term is positive. When $k=3$ the term is negative BUT SMALLER (in absolute value) THAN THE $k=2$ TERM.



    .....



    Get it?






    share|cite|improve this answer









    $endgroup$













    • $begingroup$
      sorry, I did not write question correctly. Now, I have corrected that. By looking at your answer I realized my mistake. Thanks
      $endgroup$
      – Hitendra Kumar
      1 hour ago














    3












    3








    3





    $begingroup$

    When $k=0$ the term is positive. When $k=1$ the term is negative BUT SMALLER (in absolute value) THAN THE $k=0$ TERM.



    When $k=2$ the term is positive. When $k=3$ the term is negative BUT SMALLER (in absolute value) THAN THE $k=2$ TERM.



    .....



    Get it?






    share|cite|improve this answer









    $endgroup$



    When $k=0$ the term is positive. When $k=1$ the term is negative BUT SMALLER (in absolute value) THAN THE $k=0$ TERM.



    When $k=2$ the term is positive. When $k=3$ the term is negative BUT SMALLER (in absolute value) THAN THE $k=2$ TERM.



    .....



    Get it?







    share|cite|improve this answer












    share|cite|improve this answer



    share|cite|improve this answer










    answered 1 hour ago









    David G. StorkDavid G. Stork

    11.1k41432




    11.1k41432












    • $begingroup$
      sorry, I did not write question correctly. Now, I have corrected that. By looking at your answer I realized my mistake. Thanks
      $endgroup$
      – Hitendra Kumar
      1 hour ago


















    • $begingroup$
      sorry, I did not write question correctly. Now, I have corrected that. By looking at your answer I realized my mistake. Thanks
      $endgroup$
      – Hitendra Kumar
      1 hour ago
















    $begingroup$
    sorry, I did not write question correctly. Now, I have corrected that. By looking at your answer I realized my mistake. Thanks
    $endgroup$
    – Hitendra Kumar
    1 hour ago




    $begingroup$
    sorry, I did not write question correctly. Now, I have corrected that. By looking at your answer I realized my mistake. Thanks
    $endgroup$
    – Hitendra Kumar
    1 hour ago











    0












    $begingroup$

    We can specifically prove that
    $$
    boxed{sum_{k=0}^{n} {n choose k}frac{{(-1)}^k}{n+k+1}=left((2n+1)binom{2n}nright)^{-1}}
    $$

    To see this, shuffle a deck of $2n+1$ cards numbered $1$ to $n$. Consider this:




    What is the probability that card number $n+1$ is in the middle of the deck, and cards numbered $1$ to $n$ are below it?




    The easy answer is the fraction on the RHS. The LHS can be interpreted as an application of the principle of inclusion exclusion. Namely, we first take the probability that card number of $n+1$ is the lowest of the cards numbered $n+1,n+2,dots,2n+1$. This is the $k=0$ term. From this, for each $i=1,dots,n$, we subtract the probability that $n+1$ is the lowest of the list $i,n+1,n+2,dots,2n+1$. This is a bad event, because we want $n+1$ to be above $i$. Doing this for each $i$, we subtract $binom{n}1frac{1}{n+2}$. We then must add back in the doubly subtracted events, subtract the triple intersections, and so on, eventually ending with the alternating sum on the left.






    share|cite









    $endgroup$


















      0












      $begingroup$

      We can specifically prove that
      $$
      boxed{sum_{k=0}^{n} {n choose k}frac{{(-1)}^k}{n+k+1}=left((2n+1)binom{2n}nright)^{-1}}
      $$

      To see this, shuffle a deck of $2n+1$ cards numbered $1$ to $n$. Consider this:




      What is the probability that card number $n+1$ is in the middle of the deck, and cards numbered $1$ to $n$ are below it?




      The easy answer is the fraction on the RHS. The LHS can be interpreted as an application of the principle of inclusion exclusion. Namely, we first take the probability that card number of $n+1$ is the lowest of the cards numbered $n+1,n+2,dots,2n+1$. This is the $k=0$ term. From this, for each $i=1,dots,n$, we subtract the probability that $n+1$ is the lowest of the list $i,n+1,n+2,dots,2n+1$. This is a bad event, because we want $n+1$ to be above $i$. Doing this for each $i$, we subtract $binom{n}1frac{1}{n+2}$. We then must add back in the doubly subtracted events, subtract the triple intersections, and so on, eventually ending with the alternating sum on the left.






      share|cite









      $endgroup$
















        0












        0








        0





        $begingroup$

        We can specifically prove that
        $$
        boxed{sum_{k=0}^{n} {n choose k}frac{{(-1)}^k}{n+k+1}=left((2n+1)binom{2n}nright)^{-1}}
        $$

        To see this, shuffle a deck of $2n+1$ cards numbered $1$ to $n$. Consider this:




        What is the probability that card number $n+1$ is in the middle of the deck, and cards numbered $1$ to $n$ are below it?




        The easy answer is the fraction on the RHS. The LHS can be interpreted as an application of the principle of inclusion exclusion. Namely, we first take the probability that card number of $n+1$ is the lowest of the cards numbered $n+1,n+2,dots,2n+1$. This is the $k=0$ term. From this, for each $i=1,dots,n$, we subtract the probability that $n+1$ is the lowest of the list $i,n+1,n+2,dots,2n+1$. This is a bad event, because we want $n+1$ to be above $i$. Doing this for each $i$, we subtract $binom{n}1frac{1}{n+2}$. We then must add back in the doubly subtracted events, subtract the triple intersections, and so on, eventually ending with the alternating sum on the left.






        share|cite









        $endgroup$



        We can specifically prove that
        $$
        boxed{sum_{k=0}^{n} {n choose k}frac{{(-1)}^k}{n+k+1}=left((2n+1)binom{2n}nright)^{-1}}
        $$

        To see this, shuffle a deck of $2n+1$ cards numbered $1$ to $n$. Consider this:




        What is the probability that card number $n+1$ is in the middle of the deck, and cards numbered $1$ to $n$ are below it?




        The easy answer is the fraction on the RHS. The LHS can be interpreted as an application of the principle of inclusion exclusion. Namely, we first take the probability that card number of $n+1$ is the lowest of the cards numbered $n+1,n+2,dots,2n+1$. This is the $k=0$ term. From this, for each $i=1,dots,n$, we subtract the probability that $n+1$ is the lowest of the list $i,n+1,n+2,dots,2n+1$. This is a bad event, because we want $n+1$ to be above $i$. Doing this for each $i$, we subtract $binom{n}1frac{1}{n+2}$. We then must add back in the doubly subtracted events, subtract the triple intersections, and so on, eventually ending with the alternating sum on the left.







        share|cite












        share|cite



        share|cite










        answered 9 mins ago









        Mike EarnestMike Earnest

        25.5k22151




        25.5k22151






























            draft saved

            draft discarded




















































            Thanks for contributing an answer to Mathematics Stack Exchange!


            • Please be sure to answer the question. Provide details and share your research!

            But avoid



            • Asking for help, clarification, or responding to other answers.

            • Making statements based on opinion; back them up with references or personal experience.


            Use MathJax to format equations. MathJax reference.


            To learn more, see our tips on writing great answers.




            draft saved


            draft discarded














            StackExchange.ready(
            function () {
            StackExchange.openid.initPostLogin('.new-post-login', 'https%3a%2f%2fmath.stackexchange.com%2fquestions%2f3157740%2fshowing-a-sum-is-positive%23new-answer', 'question_page');
            }
            );

            Post as a guest















            Required, but never shown





















































            Required, but never shown














            Required, but never shown












            Required, but never shown







            Required, but never shown

































            Required, but never shown














            Required, but never shown












            Required, but never shown







            Required, but never shown







            Popular posts from this blog

            “%fieldName is a required field.”, in Magento2 REST API Call for GET Method Type The Next...

            How to change City field to a dropdown in Checkout step Magento 2Magento 2 : How to change UI field(s)...

            夢乃愛華...